How to check if this function is continuous

Click For Summary
To determine if a function f(z) is continuous at z=0, one must evaluate the limits along different paths. If the limits yield different values, as shown in the discussion, the limit does not exist, indicating that f(z) is not continuous at that point. Even if both limits approach the same absolute value but differ in sign, the limit still does not exist, confirming discontinuity. Therefore, continuity at z=0 requires identical limits from all paths. The discussion emphasizes the importance of consistent limit values for establishing continuity.
MissP.25_5
Messages
329
Reaction score
0
Hello.
The question is in the attached, together with my attempt. As you can see, I found the limit, but I don't know what each value means. If I have calculated the limits correctly, how do I know know if f(z) is continuous at 0 or not?
 

Attachments

  • IMG_6392.jpg
    IMG_6392.jpg
    19.9 KB · Views: 510
Physics news on Phys.org
You have two different answers for the limit along different paths so the limit does not exist, therefore f(z) is not continuous at z=0.
 
benorin said:
You have two different answers for the limit along different paths so the limit does not exist, therefore f(z) is not continuous at z=0.

So, if no.1 and no.2 both had +/-1 as limits, then the function would be continuous at 0?
 
MissP.25_5 said:
So, if no.1 and no.2 both had +/-1 as limits, then the function would be continuous at 0?
No, if one is positive 1 and the other is negative 1, they are still different and thus the limit d.n.e. So it's not continuous there.
 
benorin said:
No, if one is positive 1 and the other is negative 1, they are still different and thus the limit d.n.e. So it's not continuous there.

Got it, thanks! By the way, could you look at my thread entitled contour integral please?
 
Question: A clock's minute hand has length 4 and its hour hand has length 3. What is the distance between the tips at the moment when it is increasing most rapidly?(Putnam Exam Question) Answer: Making assumption that both the hands moves at constant angular velocities, the answer is ## \sqrt{7} .## But don't you think this assumption is somewhat doubtful and wrong?

Similar threads

  • · Replies 11 ·
Replies
11
Views
2K
  • · Replies 3 ·
Replies
3
Views
1K
  • · Replies 27 ·
Replies
27
Views
2K
  • · Replies 8 ·
Replies
8
Views
3K
  • · Replies 7 ·
Replies
7
Views
2K
Replies
7
Views
1K
Replies
4
Views
4K
Replies
4
Views
2K
Replies
3
Views
1K
  • · Replies 2 ·
Replies
2
Views
1K